Mittag-Leffler condition and the vanishing of \(\varprojlim^ 1\) (Q1910451): Difference between revisions

From MaRDI portal
Import240304020342 (talk | contribs)
Set profile property.
Set OpenAlex properties.
 
Property / full work available at URL
 
Property / full work available at URL: https://doi.org/10.1016/0040-9383(94)00056-5 / rank
 
Normal rank
Property / OpenAlex ID
 
Property / OpenAlex ID: W2081589340 / rank
 
Normal rank

Latest revision as of 02:53, 20 March 2024

scientific article
Language Label Description Also known as
English
Mittag-Leffler condition and the vanishing of \(\varprojlim^ 1\)
scientific article

    Statements

    Mittag-Leffler condition and the vanishing of \(\varprojlim^ 1\) (English)
    0 references
    0 references
    0 references
    8 April 1996
    0 references
    The author considers inverse limits \(\varprojlim\) of (right) modules over a fixed ring \(k\), with indexing category the natural numbers \(\mathbb{N}\). An inverse system \(\Xi= (X_m)_{m\in\mathbb{N}}\) satisfies the Mittag-Leffler condition if for all \(n\) the decreasing filtration of \(X_n\) formed by the images of \(X_m\) for \(m>n\) is eventually constant. The main results are the following: an inverse system \(\Xi\) satisfies the Mittag-Leffler condition if and only if \(\varprojlim^1 (\Xi\otimes E)=0\) for any left \(k\)-module \(E\) if and only if \(\varprojlim^1 (\Xi \otimes K)=0\) for any ring extension \(K\) of \(k\).
    0 references
    inverse limits
    0 references
    Mittag-Leffler condition
    0 references

    Identifiers